Happy Memorial Day! Please note that AoPS Online is closed May 24-26th.

G
Topic
First Poster
Last Poster
Integers on a cube
Rushil   6
N an hour ago by SomeonecoolLovesMaths
Source: Indian RMO 2004 Problem 2
Positive integers are written on all the faces of a cube, one on each. At each corner of the cube, the product of the numbers on the faces that meet at the vertex is written. The sum of the numbers written on the corners is 2004. If T denotes the sum of the numbers on all the faces, find the possible values of T.
6 replies
Rushil
Feb 28, 2006
SomeonecoolLovesMaths
an hour ago
36x⁴ + 12x² - 36x + 13 > 0
fxandi   4
N Today at 10:22 AM by wh0nix
Prove that for any real $x \geq 0$ holds inequality $36x^4 + 12x^2 - 36x + 13 > 0.$
4 replies
fxandi
May 5, 2025
wh0nix
Today at 10:22 AM
2024 Miklós-Schweitzer problem 3
Martin.s   3
N Today at 1:30 AM by naenaendr
Do there exist continuous functions $f, g: \mathbb{R} \to \mathbb{R}$, both nowhere differentiable, such that $f \circ g$ is differentiable?
3 replies
Martin.s
Dec 5, 2024
naenaendr
Today at 1:30 AM
Putnam 2017 B3
goveganddomath   35
N Yesterday at 9:43 PM by Kempu33334
Source: Putnam
Suppose that $$f(x) = \sum_{i=0}^\infty c_ix^i$$is a power series for which each coefficient $c_i$ is $0$ or $1$. Show that if $f(2/3) = 3/2$, then $f(1/2)$ must be irrational.
35 replies
goveganddomath
Dec 3, 2017
Kempu33334
Yesterday at 9:43 PM
Putnam 2003 B3
btilm305   31
N Yesterday at 9:41 PM by Kempu33334
Show that for each positive integer n, \[n!=\prod_{i=1}^n \; \text{lcm} \; \{1, 2, \ldots, \left\lfloor\frac{n}{i} \right\rfloor\}\](Here lcm denotes the least common multiple, and $\lfloor x\rfloor$ denotes the greatest integer $\le x$.)
31 replies
btilm305
Jun 23, 2011
Kempu33334
Yesterday at 9:41 PM
sequence of highly correlated rv pairs
Konigsberg   0
Yesterday at 8:12 PM
For a given $0 < \varepsilon< 1$, construct a sequence of random variables $X_1,\dots,X_n$ such that
$$
\operatorname{Corr}(X_i,X_{i+1}) = 1-\varepsilon,\qquad 1\le i\le n-1.
$$Let
$$
f(\varepsilon)=\min\left\{\,n\in\mathbb N \mid \text{it is possible that }\operatorname{Corr}(X_1,X_n)<0\right\}.
$$Find positive constants $c_1$ and $c_2$ such that
$$
\lim_{\varepsilon\to0}\frac{f(\varepsilon)}{c_1\,\varepsilon^{c_2}}=1.
$$
0 replies
Konigsberg
Yesterday at 8:12 PM
0 replies
Group theory with unexpected NT example
AndreiVila   2
N Yesterday at 1:31 PM by CHOUKRI
Source: Romanian District Olympiad 2025 12.1
Let $G$ be a group and $A$ a nonempty subset of $G$. Let $AA=\{xy\mid x,y\in A\}$.
[list=a]
[*] Prove that if $G$ is finite, then $AA=A$ if and only if $|A|=|AA|$ and $e\in A$.
[*] Give an example of a group $G$ and a nonempty subset $A$ of $G$ such that $AA\neq A$, $|AA|=|A|$ and $AA$ is a proper subgroup of $G$.
[/list]
Mathematical Gazette - Robert Rogozsan
2 replies
AndreiVila
Mar 8, 2025
CHOUKRI
Yesterday at 1:31 PM
random variables
ORguy   2
N Yesterday at 1:06 PM by OGMATH
Source: traffic accidents
The time (in hours) between consecutive traffic accidents can be described by the exponential random variable X with parameter f=1/60.

Find

P(X <= 60),

P(X >120), and

P(10 < X <= 100)
2 replies
ORguy
Nov 29, 2007
OGMATH
Yesterday at 1:06 PM
Reducing the exponents for good
RobertRogo   1
N Yesterday at 11:31 AM by RobertRogo
Source: The national Algebra contest (Romania), 2025, Problem 3/Abstract Algebra (a bit generalized)
Let $A$ be a ring with unity such that for every $x \in A$ there exist $t_x, n_x \in \mathbb{N}^*$ such that $x^{t_x+n_x}=x^{n_x}$. Prove that
a) If $t_x \cdot 1 \in U(A), \forall x \in A$ then $x^{t_x+1}=x, \forall x \in A$
b) If there is an $x \in A$ such that $t_x \cdot 1 \notin U(A)$ then the result from a) may no longer hold.

Authors: Laurențiu Panaitopol, Dorel Miheț, Mihai Opincariu, me, Filip Munteanu
1 reply
RobertRogo
May 20, 2025
RobertRogo
Yesterday at 11:31 AM
Convergence of complex sequence
Rohit-2006   9
N Yesterday at 3:41 AM by Saucitom
Suppose $z_1, z_2,\cdots,z_k$ are complex numbers with absolute value $1$. For $n=1,2,\cdots$ define $w_n=z_1^n+z_2^n+\cdots+z_k^n$. Given that the sequence $(w_n)_{n\geq1}$ converges. Show that,
$$z_1=z_2=\cdots=z_k=1$$.
9 replies
Rohit-2006
May 17, 2025
Saucitom
Yesterday at 3:41 AM
Invertible Matrices
Mateescu Constantin   8
N May 22, 2025 by loup blanc
Source: Romanian District Olympiad 2018 - Grade XI - Problem 1
Show that if $n\ge 2$ is an integer, then there exist invertible matrices $A_1, A_2, \ldots, A_n \in \mathcal{M}_2(\mathbb{R})$ with non-zero entries such that:

\[A_1^{-1} + A_2^{-1} + \ldots + A_n^{-1} = (A_1 + A_2 + \ldots + A_n)^{-1}.\]
Edit.
8 replies
Mateescu Constantin
Mar 10, 2018
loup blanc
May 22, 2025
Squares on height in right triangle
Miquel-point   1
N Apr 20, 2025 by LiamChen
Source: Romanian NMO 2025 7.4
Consider the right-angled triangle $ABC$ with $\angle A$ right and $AD\perp BC$, $D\in BC$. On the ray $[AD$ we take two points $E$ and $H$ so that $AE=AC$ and $AH=AB$. Consider the squares $AEFG$ and $AHJI$ containing inside $C$ and $B$, respectively. If $K=EG\cap AC$ and $L=IH\cap AB$, $N=IL\cap GK$ and $M=IB\cap GC$, prove that $LK\parallel BC$ and that $A$, $N$ and $M$ are collinear.
1 reply
Miquel-point
Apr 19, 2025
LiamChen
Apr 20, 2025
Squares on height in right triangle
G H J
G H BBookmark kLocked kLocked NReply
Source: Romanian NMO 2025 7.4
The post below has been deleted. Click to close.
This post has been deleted. Click here to see post.
Miquel-point
499 posts
#1
Y by
Consider the right-angled triangle $ABC$ with $\angle A$ right and $AD\perp BC$, $D\in BC$. On the ray $[AD$ we take two points $E$ and $H$ so that $AE=AC$ and $AH=AB$. Consider the squares $AEFG$ and $AHJI$ containing inside $C$ and $B$, respectively. If $K=EG\cap AC$ and $L=IH\cap AB$, $N=IL\cap GK$ and $M=IB\cap GC$, prove that $LK\parallel BC$ and that $A$, $N$ and $M$ are collinear.
Z K Y
The post below has been deleted. Click to close.
This post has been deleted. Click here to see post.
LiamChen
34 posts
#2
Y by
My solution:
Attachments:
Z K Y
N Quick Reply
G
H
=
a